3
$\begingroup$

I have a concern about a result given by Murty in [1] and also written by Floudas and Visweswaran in [2]

They consider a QP:

\begin{array}{ll}{\min _{x} Q(x)} & {=c^{T} x+\frac{1}{2} x^{T} D x} \\ {\text {s.t.} \quad A x} & {\geq b} \\ {x} & {\geq 0}\end{array}

with no particular assumption on $D$ (appart from symmetric). Murty argues that if $x_{*}$ is a solution of the QP it is also solution of the following LP:

\begin{array}{cl}{\min _{x}} & {\left(c+x_{*}^{T} D\right) x} \\ {\text {s.t.}} & {A x \geq b} \\ {x} & {\geq 0}\end{array}

The proof of Murty is the following:

Let $\hat{x}$ be any feasible solution of the LP and $x_{\lambda}=\lambda \hat{x}+(1-\lambda) x_{*}$ for $\lambda \in ]0,1[$. It is also feasible since the set of feasible solutions is a convex polyhedron. Then by optimality of $x_{*}, Q\left(x_{\lambda}\right)-Q(x_{*}) \geq 0$ which implies:

$$\lambda\left(c+x_{*}^{T} D\right)(\hat{x}-x_{*})+(1 / 2)\lambda^{2}(\hat{x}-x_{*})^{T} D(\hat{x}-x_{*}) \geq 0 $$

And so dividing by $\lambda$ leads to:

$$\left(c+x_{*}^{T} D\right)(\hat{x}-x_{*}) \geq(-\lambda / 2)(\hat{x}-x_{*})^{T} D(\hat{x}-x_{*})$$

At this part Murty argues that it "obviously implies":

$$\left(c+x_{*}^{T} D\right)(\hat{x}-x_{*}) \geq 0 $$

and hence $\hat{x}$ must be an optimum feasible solution of the LP.

It seems to be false at first sight except when the QP is concave (and D neg definite) so that the rightmost term of the inequality is negative. Am I missing something ?

Ref:

[1] Linear Complementarity, Linear and non Linear Programming, 1988

[2] Quadratic Optimization by Floudas and Visweswaran, 1995

$\endgroup$
5
  • $\begingroup$ Murty is correct. Take the limit of the inequality for $(c + x_*^\top D)(\hat{x} - x_*)$ as $\lambda \to 0+$. $\endgroup$ Commented Jan 16, 2020 at 17:47
  • 1
    $\begingroup$ $Ax \le b$ or $Ax \ge b$? $\endgroup$ Commented Jan 16, 2020 at 18:04
  • $\begingroup$ But if it is correct then would it imply that any solution of a QP lies in an extremal point of the polyhedron ? This is true for concave QP but in general it seems strong right ? In [2] it is $\leq$ for the definition of the QP and then $\geq$ for the theorem and in the original [1] it is $\geq$ everywhere. I edit my post $\endgroup$ Commented Jan 16, 2020 at 18:25
  • $\begingroup$ No it does not imply an optimal solution is an extreme point. For example, the optimal solution of the QP could be an interior point of the feasible region, in which case the objective of the LP is $0$, and every point of the feasible region is optimal for the LP. $\endgroup$ Commented Jan 16, 2020 at 18:54
  • $\begingroup$ thank you very much for your answers I got it! $\endgroup$ Commented Jan 16, 2020 at 20:50

0

You must log in to answer this question.

Start asking to get answers

Find the answer to your question by asking.

Ask question

Explore related questions

See similar questions with these tags.